Difference between revisions of "2021 AIME II Problems/Problem 9"
(→Problem) |
MRENTHUSIASM (talk | contribs) (→Claim 1 (GCD Property): Although it is good to prove this "obvious" property, the proof is more like using the fact that gcd(x,z)=1 and gcd(y,z)=1 imply gcd(xy,z)=1. This is using a simpler version of the claim to prove the claim itself.) |
||
(87 intermediate revisions by 6 users not shown) | |||
Line 2: | Line 2: | ||
Find the number of ordered pairs <math>(m, n)</math> such that <math>m</math> and <math>n</math> are positive integers in the set <math>\{1, 2, ..., 30\}</math> and the greatest common divisor of <math>2^m + 1</math> and <math>2^n - 1</math> is not <math>1</math>. | Find the number of ordered pairs <math>(m, n)</math> such that <math>m</math> and <math>n</math> are positive integers in the set <math>\{1, 2, ..., 30\}</math> and the greatest common divisor of <math>2^m + 1</math> and <math>2^n - 1</math> is not <math>1</math>. | ||
− | ==Solution== | + | ==Solution 1== |
− | + | This solution refers to the <b>Remarks</b> section. | |
− | ==See | + | By the Euclidean Algorithm, we have <cmath>\gcd\left(2^m+1,2^m-1\right)=\gcd\left(2,2^m-1\right)=1.</cmath> |
+ | We are given that <math>\gcd\left(2^m+1,2^n-1\right)>1.</math> Multiplying both sides by <math>\gcd\left(2^m-1,2^n-1\right)</math> gives | ||
+ | <cmath>\begin{align*} | ||
+ | \gcd\left(2^m+1,2^n-1\right)\cdot\gcd\left(2^m-1,2^n-1\right)&>\gcd\left(2^m-1,2^n-1\right) \\ | ||
+ | \gcd\left(\left(2^m+1\right)\left(2^m-1\right),2^n-1\right)&>\gcd\left(2^m-1,2^n-1\right) \hspace{12mm} &&\text{by }\textbf{Claim 1} \\ | ||
+ | \gcd\left(2^{2m}-1,2^n-1\right)&>\gcd\left(2^m-1,2^n-1\right) \\ | ||
+ | 2^{\gcd(2m,n)}-1&>2^{\gcd(m,n)}-1 &&\text{by }\textbf{Claim 2} \\ | ||
+ | \gcd(2m,n)&>\gcd(m,n), | ||
+ | \end{align*}</cmath> | ||
+ | which implies that <math>n</math> must have more factors of <math>2</math> than <math>m</math> does. | ||
+ | |||
+ | We construct the following table for the first <math>30</math> positive integers: | ||
+ | <cmath>\begin{array}{c|c|c} | ||
+ | && \\ [-2.5ex] | ||
+ | \boldsymbol{\#}\textbf{ of Factors of }\boldsymbol{2} & \textbf{Numbers} & \textbf{Count} \\ | ||
+ | \hline | ||
+ | && \\ [-2.25ex] | ||
+ | 0 & 1,3,5,7,9,11,13,15,17,19,21,23,25,27,29 & 15 \\ | ||
+ | && \\ [-2.25ex] | ||
+ | 1 & 2,6,10,14,18,22,26,30 & 8 \\ | ||
+ | && \\ [-2.25ex] | ||
+ | 2 & 4,12,20,28 & 4 \\ | ||
+ | && \\ [-2.25ex] | ||
+ | 3 & 8,24 & 2 \\ | ||
+ | && \\ [-2.25ex] | ||
+ | 4 & 16 & 1 \\ | ||
+ | \end{array}</cmath> | ||
+ | To count the ordered pairs <math>(m,n),</math> we perform casework on the number of factors of <math>2</math> that <math>m</math> has: | ||
+ | <ol style="margin-left: 1.5em;"> | ||
+ | <li>If <math>m</math> has <math>0</math> factors of <math>2,</math> then <math>m</math> has <math>15</math> options and <math>n</math> has <math>8+4+2+1=15</math> options. So, this case has <math>15\cdot15=225</math> ordered pairs.</li><p> | ||
+ | <li>If <math>m</math> has <math>1</math> factor of <math>2,</math> then <math>m</math> has <math>8</math> options and <math>n</math> has <math>4+2+1=7</math> options. So, this case has <math>8\cdot7=56</math> ordered pairs.</li><p> | ||
+ | <li>If <math>m</math> has <math>2</math> factors of <math>2,</math> then <math>m</math> has <math>4</math> options and <math>n</math> has <math>2+1=3</math> options. So, this case has <math>4\cdot3=12</math> ordered pairs.</li><p> | ||
+ | <li>If <math>m</math> has <math>3</math> factors of <math>2,</math> then <math>m</math> has <math>2</math> options and <math>n</math> has <math>1</math> option. So, this case has <math>2\cdot1=2</math> ordered pairs.</li> | ||
+ | </ol> | ||
+ | Together, the answer is <math>225+56+12+2=\boxed{295}.</math> | ||
+ | |||
+ | ~Lcz ~MRENTHUSIASM | ||
+ | |||
+ | ==Solution 2== | ||
+ | |||
+ | Consider any ordered pair <math>(m,n)</math> such that <math>\gcd(2^m+1, 2^n-1) > 1</math>. There must exist some odd number <math>p\ne 1</math> such that <math>2^m \equiv -1 \pmod{p}</math> and <math>2^n \equiv 1 \pmod{p}</math>. Let <math>d</math> be the order of <math>2</math> modulo <math>p</math>. Note that <math>2^{2m} \equiv 1 \pmod{p}</math>. From this, we can say that <math>2m</math> and <math>n</math> are both multiples of <math>d</math>, but <math>m</math> is not. Thus, we have <math>v_2(n) \ge v_2(d)</math> and <math>v_2(m) + 1 = v_2(d)</math>. Substituting the latter equation into the inequality before gives <math>v_2(n) \ge v_2(m)+1</math>. Since <math>v_2(n)</math> and <math>v_2(m)</math> are integers, this implies <math>v_2(n)>v_2(m)</math>. The rest of the solution now proceeds as in Solution 1. | ||
+ | |||
+ | ~Sedro | ||
+ | |||
+ | ==Remarks== | ||
+ | |||
+ | ===Claim 1 (GCD Property)=== | ||
+ | <b>If <math>\boldsymbol{r,s,}</math> and <math>\boldsymbol{t}</math> are positive integers such that <math>\boldsymbol{\gcd(r,s)=1,}</math> then <math>\boldsymbol{\gcd(r,t)\cdot\gcd(s,t)=\gcd(rs,t).}</math></b> | ||
+ | |||
+ | As <math>r</math> and <math>s</math> are relatively prime (have no prime divisors in common), this property is intuitive. | ||
+ | |||
+ | ~MRENTHUSIASM | ||
+ | |||
+ | ===Claim 2 (Olympiad Number Theory Lemma)=== | ||
+ | <b>If <math>\boldsymbol{u,a,}</math> and <math>\boldsymbol{b}</math> are positive integers such that <math>\boldsymbol{u\geq2,}</math> then <math>\boldsymbol{\gcd\left(u^a-1,u^b-1\right)=u^{\gcd(a,b)}-1.}</math></b> | ||
+ | |||
+ | There are two proofs to this claim, as shown below. | ||
+ | |||
+ | ~MRENTHUSIASM | ||
+ | |||
+ | ===Claim 2 Proof 1 (Euclidean Algorithm)=== | ||
+ | If <math>a=b,</math> then <math>\gcd(a,b)=a=b,</math> from which the claim is clearly true. | ||
+ | |||
+ | Otherwise, let <math>a>b</math> without the loss of generality. For all integers <math>p</math> and <math>q</math> such that <math>p>q>0,</math> the Euclidean Algorithm states that <cmath>\gcd(p,q)=\gcd(p-q,q)=\cdots=\gcd(p\operatorname{mod}q,q).</cmath> We apply this result repeatedly to reduce the larger number: <cmath>\gcd\left(u^a-1,u^b-1\right)=\gcd\left(u^a-1-u^{a-b}\left(u^b-1\right),u^b-1\right)=\gcd\left(u^{a-b}-1,u^b-1\right).</cmath> | ||
+ | Continuing, we have | ||
+ | <cmath>\begin{align*} | ||
+ | \gcd\left(u^a-1,u^b-1\right)&=\gcd\left(u^{a-b}-1,u^b-1\right) \\ | ||
+ | & \ \vdots \\ | ||
+ | &=\gcd\left(u^{\gcd(a,b)}-1,u^{\gcd(a,b)}-1\right) \\ | ||
+ | &=u^{\gcd(a,b)}-1, | ||
+ | \end{align*}</cmath> | ||
+ | from which the proof is complete. | ||
+ | |||
+ | ~MRENTHUSIASM | ||
+ | |||
+ | ===Claim 2 Proof 2 (Bézout's Identity)=== | ||
+ | Let <math>d=\gcd\left(u^a-1,u^b-1\right).</math> It follows that <math>u^a\equiv1\pmod{d}</math> and <math>u^b\equiv1\pmod{d}.</math> | ||
+ | |||
+ | By Bézout's Identity, there exist integers <math>x</math> and <math>y</math> such that <math>ax+by=\gcd(a,b),</math> so <cmath>u^{\gcd(a,b)}=u^{ax+by}=(u^a)^x\cdot(u^b)^y\equiv1\pmod{d},</cmath> | ||
+ | from which <math>u^{\gcd(a,b)}-1\equiv0\pmod{d}.</math> We know that <math>u^{\gcd(a,b)}-1\geq d.</math> | ||
+ | |||
+ | Next, we notice that | ||
+ | <cmath>\begin{align*} | ||
+ | u^a-1&=\left(u^{\gcd(a,b)}-1\right)\left(u^{a-\gcd{(a,b)}}+u^{a-2\gcd{(a,b)}}+u^{a-3\gcd{(a,b)}}+\cdots+1\right), \\ | ||
+ | u^b-1&=\left(u^{\gcd(a,b)}-1\right)\left(u^{b-\gcd{(a,b)}}+u^{b-2\gcd{(a,b)}}+u^{b-3\gcd{(a,b)}}+\cdots+1\right). | ||
+ | \end{align*}</cmath> | ||
+ | Since <math>u^{\gcd(a,b)}-1</math> is a common divisor of <math>u^a-1</math> and <math>u^b-1,</math> we conclude that <math>u^{\gcd(a,b)}-1=d,</math> from which the proof is complete. | ||
+ | |||
+ | ~MRENTHUSIASM | ||
+ | |||
+ | ==Video Solution== | ||
+ | https://youtu.be/h3awf7yhGZ4 | ||
+ | |||
+ | ~MathProblemSolvingSkills.com | ||
+ | |||
+ | ==Video Solution by Interstigation== | ||
+ | https://youtu.be/kasgsb0Rge4 | ||
+ | |||
+ | ~Interstigation | ||
+ | |||
+ | ==See Also== | ||
{{AIME box|year=2021|n=II|num-b=8|num-a=10}} | {{AIME box|year=2021|n=II|num-b=8|num-a=10}} | ||
{{MAA Notice}} | {{MAA Notice}} |
Latest revision as of 00:49, 1 November 2024
Contents
Problem
Find the number of ordered pairs such that and are positive integers in the set and the greatest common divisor of and is not .
Solution 1
This solution refers to the Remarks section.
By the Euclidean Algorithm, we have We are given that Multiplying both sides by gives which implies that must have more factors of than does.
We construct the following table for the first positive integers: To count the ordered pairs we perform casework on the number of factors of that has:
- If has factors of then has options and has options. So, this case has ordered pairs.
- If has factor of then has options and has options. So, this case has ordered pairs.
- If has factors of then has options and has options. So, this case has ordered pairs.
- If has factors of then has options and has option. So, this case has ordered pairs.
Together, the answer is
~Lcz ~MRENTHUSIASM
Solution 2
Consider any ordered pair such that . There must exist some odd number such that and . Let be the order of modulo . Note that . From this, we can say that and are both multiples of , but is not. Thus, we have and . Substituting the latter equation into the inequality before gives . Since and are integers, this implies . The rest of the solution now proceeds as in Solution 1.
~Sedro
Remarks
Claim 1 (GCD Property)
If and are positive integers such that then
As and are relatively prime (have no prime divisors in common), this property is intuitive.
~MRENTHUSIASM
Claim 2 (Olympiad Number Theory Lemma)
If and are positive integers such that then
There are two proofs to this claim, as shown below.
~MRENTHUSIASM
Claim 2 Proof 1 (Euclidean Algorithm)
If then from which the claim is clearly true.
Otherwise, let without the loss of generality. For all integers and such that the Euclidean Algorithm states that We apply this result repeatedly to reduce the larger number: Continuing, we have from which the proof is complete.
~MRENTHUSIASM
Claim 2 Proof 2 (Bézout's Identity)
Let It follows that and
By Bézout's Identity, there exist integers and such that so from which We know that
Next, we notice that Since is a common divisor of and we conclude that from which the proof is complete.
~MRENTHUSIASM
Video Solution
~MathProblemSolvingSkills.com
Video Solution by Interstigation
~Interstigation
See Also
2021 AIME II (Problems • Answer Key • Resources) | ||
Preceded by Problem 8 |
Followed by Problem 10 | |
1 • 2 • 3 • 4 • 5 • 6 • 7 • 8 • 9 • 10 • 11 • 12 • 13 • 14 • 15 | ||
All AIME Problems and Solutions |
The problems on this page are copyrighted by the Mathematical Association of America's American Mathematics Competitions.